An ice cream shop has 24 flavors. Anne wants to buy a three scoop cone with three different flavors. How many cones could she buy if the order of the flavors is important.

Answers

Answer 1

If Anne wants to buy a three-scoop cone with three different flavors, and the order of the flavors is important, she has 24 choices for the first scoop, 23 choices for the second scoop (since it must be different from the first), and 22 choices for the third scoop (since it must be different from the first two).

To calculate the total number of cones she could buy, we multiply the number of choices for each scoop:

Number of cones = 24 * 23 * 22

Number of cones = 12,144

Therefore, Anne could buy 12,144 different cones with three different flavors, considering the order of the flavors is important.

I hope this helps! :)

Related Questions

The measure of one small angles of a right triangle is 45 less than twice the measure of the other small angle. Find the measure of both angles

Answers

Answer:

x + x - 45 = 90

2x - 45 = 90

2x = 135

x = 67.5, so x - 45 = 22.5

The other two angles measure 22.5° and 67.5°.

Find the Surface Area of the following figure. 9.5 m 16m 14m 12.7m 11m

Answers

The total surface area of the figure will be 1968.85 square meters.

To determine  the surface area of the figure, we need to find the area of each face and then add them together.

Surface Area of the rectangular prism = 2(lb + bh + hl)

= 2(16 × 9.5) + 2(9.5 × 14) + 2(16 × 14)

= 2(152 + 133 + 224) = 2(509)

= 1018 m²

Next, we need to find the area of the triangular prism on the front with dimensions 11 m, 12.7 m, and 14 m:

Surface Area of the triangular prism;

= (11 × 14) + 2(0.5 × 11 × 12.7) + 2(0.5 × 12.7 × 14)

= (154 + 350.35 + 445.5)

= 950.85 m²

Therefore, the total surface area of the figure will be;

Total Surface Area = Surface Area of rectangular prism + Surface Area of triangular prism

= 1018 m² + 950.85 m²

= 1968.85 m²

So, the surface area of the figure is 1968.85 square meters.

Learn more about a rectangular prism;

https://brainly.com/question/21308574

#SPJ1

Let (2,-5) be a point on the terminal side of theta find the exact values of cos theta csc theta and tan theat

Answers

Answer:

Step-by-step explanation:

x = 2, y = -5

√(2²+(-5)²) = √29

cosθ = 2/√29 = (2√29)/29

sinθ = -5/√29

cscθ = 1/sinθ = -√29/5

tanθ = sinθ/cosθ = -5/2

Let (2,-5) be a point on the terminal side of theta find the exact values of cos theta csc theta and
Let (2,-5) be a point on the terminal side of theta find the exact values of cos theta csc theta and

The values of cos theta is 2/\(\sqrt{29}\) , cosec theta is \(\sqrt{29} /5\) and tan theta is 5/2 if the point on the terminal side of theta is (2,-5).

What is pythagoras theorem?

It says that in a right angled triangle the square of the hypotenuse is equal to the sum of the square of base and square of height.

How to find values of trigonometric function?

We have been given a point (2,-5) on the terminal side. Let us plot the point on the graph and join it with origin from both ends.

By pythagoras theorem we can easily find AC=\(\sqrt{5^{2}+2^{2} }\)

AC=\(\sqrt{29}\)

With the help of the sides of the triangle we can find the value of

cos theta=2/\(\sqrt{29}\)

cosec theta=\(\sqrt{29}\)/5

tan theta=5/2

Hence the values of cos theta, cosec theta, tan theta are 2/\(\sqrt{29}\),\(\sqrt{29}\)/5,5/2 respectively.

Learn more about pythagoras theorem at https://brainly.com/question/343682

#SPJ2

Let (2,-5) be a point on the terminal side of theta find the exact values of cos theta csc theta and

The perimeter of the triangular park shown on the right is 10x - 6. What is the missing length?

The perimeter of the triangular park shown on the right is 10x - 6. What is the missing length?

Answers

Answer:

missing length = 6x - 5

Step-by-step explanation:

the perimeter is the sum of the 3 sides.

subtract the sum of the 2 given sides from the perimeter for missing side, that is

10x - 6 - (x + 1 + 3x - 2)

= 10x - 6 - (4x - 1 ) ← distribute parenthesis by - 1

= 10x - 6 - 4x + 1 ← collect like terms

= 6x - 5

missing length = 6x - 5

The step function f(x) is graphed
What is the value of f(-1)?
0-3
O-1
ОО
0 1

Answers

The value of f(-1) in the graph is 1.

What is Function?

In mathematics, a function is an expression, rule, or law that establishes a relationship between an independent variable and a dependent variable (the dependent variable). In mathematics, functions exist everywhere, and they are crucial for constructing physical links in the sciences.

The graph leads to the following finding:

The values of f(-1) for x = -1 are -1 and -3.

The function does, however, have a closed circle at f(-1) = 1.

The function has an open circle at f(-1) = -3.

A closed circle represents inclusive principles.

Hence, the graph's value for f(-1) is 1.

Learn more about functions here:

brainly.com/question/13136492

#SPJ7

The step function f(x) is graphedWhat is the value of f(-1)?0-3O-10 1

The definition of a circle uses the undefined term _______

Answers

Answer:

plane because i know what it is                                                                                                  

Answer: plane

Step-by-step explanation: I just did it

Find the consumers surplus

Find the consumers surplus

Answers

The consumer surplus is approximately $145.83.

To find the consumer surplus, we first need to find the demand function's inverse, which gives us the willingness to pay for each unit of the product. The demand function is:

D(x) = √(739 - 3x)

Setting D(x) equal to the equilibrium price of $25, we get:

25 = √(739 - 3x)

Squaring both sides, we get:

625 = 739 - 3x

Solving for x, we get:

So at a price of $25 per unit, the consumer is willing to buy 38 units per month.

Now we can calculate the consumer's surplus.

The consumer\(x = (739 - 625) / 3 = 38\) surplus is the difference between the total amount that consumers are willing to pay for a certain quantity of a good and the total amount they actually pay. In this case, the consumer's surplus can be calculated as:

\(CS = \int_0^{38} [D(x) - 25] dx\)

where D(x) is the demand function, and the integral is taken over the range of 0 to 38, which represents the quantity demanded at a price of $25 per unit.

Evaluating this integral, we get:

\(CS = \int_0^{38} [\sqrt{(739 - 3x)} - 25] dx\\\\= [1/6 (739 - 3x)^{(3/2)} - 25x]_0^{38}\\\\= \$ 145.83\)

Therefore, the consumer surplus is approximately $145.83.

Learn more about function here:

https://brainly.com/question/29633660

#SPJ1

what is the equation for line b​

what is the equation for line b

Answers

Answer:

y=1/2x+1

Step-by-step explanation:

you can literally do rise over run to get the answer


A company has issued 6.7 million shares of stock. A mutual fund owns 99,000 shares of this company's stock. What
percentage of the company's shares are owned by the mutual fund? Give your answer rounded to the nearest tenth of a
percent.

Answers

99,000 / 6,700,000 = 99 / 6700

1.4776% of the shares are owned by the mutual fund

so 1.5% if I round

please give brainliest

70 PTS will mark brainly

This is Josh’s solution for the equation x^2-6x-7=0
x^2-6x-7=0
x^2-6x=7
x^2-6x+9=7+9
(x-3)^2=16
x-3=16
x=19
x-3=-16
x=-13

explain what mistake he made on step 5 which is x-3=16

Answers

Answer:

See below.

Explanation:

easy way to solve:

\(x^2-6x-7=0\)

\(x^2+x-7x-7=0\)

\(x(x+1)-7(x+1) =0\)

\((x-7)(x+1)= 0\)

\(x = 7, -1\)

Mistake:

In step 5,

Josh should have given:

\((x-3)^2=16\)

\(x-3 = \pm \sqrt{16}\)     [ changing sides changes square sign to \(\pm\) square root ]

\(x-3 = \pm 4\)

\(x -3 = 4\), \(x-3 = -4\)

\(x = 7\), \(x = -1\)

True or False: 2(5m-n) is in simplified form?

Answers

Answer:

false

Step-by-step explanation:

Answer:

False

Step-by-step explanation:

You can still simplify by distributing 2 to 5m and n

The simplifyed version would be 10m-2n

29) Selecting a blue marble, a white marble, and then a red marble.
O a.) 4/19
Ob.) 4/91
Oc.) 3/19
Od.) 7/91

29) Selecting a blue marble, a white marble, and then a red marble.O a.) 4/19Ob.) 4/91Oc.) 3/19Od.) 7/91

Answers

Answer:

4/91

Step-by-step explanation:

There are a total of 15 marbles.  In the first selection there are 15 marbles and 6 are blue.  That would be 6/15.

Since we are not replacing marbles, we now have 14 marbles and we want to select one of the 5 white marbles.  That would be 5/14.

Lastly we now only have 13 marbles and of those we want to select one of the 4 red ones.  That would be 4/13.

Now we take these 3 fractions and multiply them together.

6/15(5/14)(4/13).  I am going to try to make this a little easier and see if I can find any factors that any top number shares with any bottom number.

I see 5 on the top shares a common factor with 15 on the bottom.  I will divide these numbers by 3, so now I have

6/3(1/14)(4/14).  I think that I can do better than that.  I see that 6 on the top and 3 on the bottom share a common factor of 3.  I will divide both of these numbers by 3.  Which will give me:

2/1(1/14)(4/13) I see a 4 on top and a 14 on the bottom.  Those are both divisible by 2 so, I will divided them both by 2 and that leaves me with

2/1(1/7)(2/13)  That is a lot easier, now I will just multiple the top numbers and multiply the bottom numbers to get

4/91

What is the product?
(3a^2b^7) (5a^3b^8)

Answers

Answer:

\(\boxed{\sf \ \ \ 15a^5b^{15} \ \ \ }\)

Step-by-step explanation:

hello,

\((3a^2b^7) (5a^3b^8)=15a^{2+3}b^{7+8}=15a^5b^{15}\)

hope this helps

The table shows a function. Is the function linear or nonlinear?

The table shows a function. Is the function linear or nonlinear?

Answers

Answer:

linear

Step-by-step explanation:

linear equations are ones that go throught the origin if i remember correctly

Planes A and B intersect.
Which describes the intersection of line m and line n?
P
m
2
O point w
O point X
Opoint Y
point Z
.
n2
X
Y
W
k
.
V
2
-

Planes A and B intersect.Which describes the intersection of line m and line n?Pm2O point wO point XOpoint

Answers

Answer:

I think about point Y ......

If planes A and B intersect, point w describes the intersection of line m and line n.

What is Coordinate system?

A coordinate system is a system that uses one or more numbers, or coordinates, to uniquely determine the position of the points or other geometric elements.

In the given figure, the planes A and B intersected.

We have to find the point where the line l and line m intersects.

The intersection of a line and a line can be the empty set, a point, or another line.

Two straight lines will intersect at a point if they are not parallel.

The point of intersection is the meeting point of two straight lines.

The lines m and line n are intersected at the point w.

Hence, if planes A and B intersect, point w describes the intersection of line m and line n.

To learn more on Coordinate system click:

https://brainly.com/question/29762404

#SPJ7

due tomorrow help please :)

due tomorrow help please :)

Answers

Answer:

68°

Step-by-step explanation:

The angles are equal to each other.  Set them equal and solve for x

5x + 18 = 8x - 12  Subtract 5x from both sides of the equation

18 = 3x - 12  Add 12 to both sides of the equation

30 = 3x  Divide both sides by 3

10 = x

Now that you have found x plug is back into  5x + 18

5(10) + 18

50 + 18 = 68

Answer:

∠ FLJ = 68°

Step-by-step explanation:

∠ EKG and ∠ FLJ are alternate exterior angles and are congruent , then

8x - 12 = 5x + 18 ( subtract 5x from both sides )

3x - 12 = 18 ( add 12 to both sides )

3x = 30 ( divide both sides by 3 )

x = 10

Then

∠ FLJ = 5x + 18 = 5(10) + 18 = 50 + 18 = 68°



An online store sells two types of speaker docks for smartphones. The higher-priced speaker dock sells for $170 and the lower-priced speaker dock sells for $90. Last week the store sold three times
as many lower-priced speaker docks as higher-priced speaker docks. Combined sales totaled $3,080. How many lower-priced speaker docks did it sell?

Answers

Answer:

21

Step-by-step explanation:

the price of the lower priced docs for that week would be 3x because the store sold 3 times as many low priced than the higher priced.

> so 90(3x) = 270x

170x + 270x = 440x

3080/440 = 7

7 high priced speakers were sold

to find out the cost of the lower ones you'd do

3x7 and get 21

What is the equation of the line that passes through the point (6,2) and has a slope of 1/3?

Answers

Answer:

\(y=\frac{1}{3}x\)

*View attached graph*

Step-by-step explanation:

Point-slope: \(y-y_1=m(x-x_1)\)

m = slope (\(\frac{1}{3}\))

point (6,2)

\(y-y_1=m(x-x_1)\)

\(y-2=\frac{1}{3}(x-6)\)

\(y-2=\frac{1}{3}x-2\)  

  \(+2\)           \(+2\)

\(y=\frac{1}{3}x\)

Hope this helps!

What is the equation of the line that passes through the point (6,2) and has a slope of 1/3?

0.09 is 1/10 as much as 0.90

Answers

The mathematical statement given as 0.09 is 1/10 as much as 0.90 is true

How to determine the true statement?

The mathematical statement is given as:

0.09 is 1/10 as much as 0.90

To determine if the above statement is true, we start by representing the statement as an equation

So, we have

0.09 = 1/10 * 0.90

Express 1/10 as decimal

So, we have

0.09 = 0.10 * 0.90

Evaluate the product

So, we have

0.09 = 0.09

The above equation is true

Hence, the mathematical statement given as 0.09 is 1/10 as much as 0.90 is true


Read more about expressions at

https://brainly.com/question/723406

#SPJ1

Possible question

Determine if the following statement is true or false

-9(x+2)=54 apply the distributive property and simplify ​

Answers

Answer:

x = -8

Step-by-step explanation:

-9 (x+2) = 54

-9x - 18 = 54

-9x = 72

x = -8

Hope this helps :)

Let me know if there are any mistakes!!

what is this question

what is this question

Answers

Answer:

The answer would be b i literay just took the thest

Step-by-step explanation:

The right answer is B

factorise x³-4x²+x+6​

Answers

The binomial factors of x³- 4x²+x+6​ are (x+2), (x+3), and (x-1).

Using the splitting and grouping the terms:

x³ + 4x² + x - 6

= x³ + 2x² + 2x² + x - 6  [Splitting 4x² = 2x² + 2x²]

= (x³ + 2x²) + (2x² + x - 6)

= x² (x + 2) + (2x² + 4x - 3x - 6)

= x² (x + 2) + [ 2x (x + 2) - 3 (x + 2)]

= x² (x + 2) + (x + 2) (2x - 3)

= (x + 2) ( x² + 2x - 3)

= (x + 2) ( x² + 3x - x - 3)

= (x + 2) [x (x + 3) - 1 (x + 3)]

= (x + 2) (x + 3) (x - 1)

Hence, the binomial factors are (x + 2), (x + 3) and (x - 1)

To learn more about factorise here,

https://brainly.com/question/10718512

https://brainly.com/question/24734894

hey whats

-\sqrt{256}

Answers

Answer:

16

Step-by-step explanation:

Answer:

-16

Step-by-step explanation:

\(\textsf{Given}:\)

\(-\sqrt{256}\)

\(\textsf{As $16 \cdot 16 = 256$, rewrite $256$ as $16^2$}:\)

\(\implies -\sqrt{16^2}\)

\(\textsf{Apply radical rule} \quad \sqrt{a^2}=a, \quad a \geq 0:\)

\(\implies -16\)

a sine function has an amplitude of 6, a period of pi, and a phase shift of pi/4 . what is the y intercept of the function?

Answers

y intercept will be 0,-6 .

Given,

Amplitude = 6

Time period = pi

Phase shift = pi/4

In trigonometry, the sine function can be defined as the ratio of the length of the opposite side(perpendicular) to that of the hypotenuse in a right-angled triangle.

The sine function is used to find the unknown angle or sides of a right angled triangle.

Mathematically,

sinФ = p/h

Now ,

Let us assume the phase shift to be on right side.

So the graph will be at negative y axis.

Thus the points will be 0 , -6 .

Know more about sine graph,

https://brainly.com/question/12015707

#SPJ1

IM IN A HURRY PLEASE HELP ME QUESTION IS DOWN BELOW WORTH 15 POINTS each

IM IN A HURRY PLEASE HELP ME QUESTION IS DOWN BELOW WORTH 15 POINTS each

Answers

If the radius of the circle is 5 m and angle subtended by arc LKM is 70° then the length of the arc LNM is 25.29 m.

Given that the radius of the circle is 5 m and angle subtended by arc LKM is 70°.

From the figure given we can estimate that the length of arc LNM can be circumference of circle subtracted by length of arc LKM.

Circulference of the circle is basically the length of whole arc of the circle. It is also know as perimeter of the circle. The formula of calculating circumference of a circle is 2πr in which r is the radius.

So,

length of arc LNM=Circumference of circle-length of arc LKM.

We know that length of an arc =Θr

in which Θ is the angle in radian.

length of arc=70*π\(/180*5\)

=70π\(/36\)

Circumference of circle=2π*5

=10π

Length of arc LNM=10π-70π\(/36\)

=(360π-70π)\(/36\)

=290π\(/36\)

Put π=3.14

=290*3.14\(/36\)

=910.6\(/36\)

=25.29m

Hence the length of arc LNM is 25.29 m.

Learn more about arc at https://brainly.com/question/2005046

#SPJ1

which inequality describes the possible values of x? there are three triangles , triangle abc, triangle cde, and triangle efg. the points a, c , e, and g are collinear, points b, c, and d are collinear, and points d, e, and f are collinear. side ac, side ce, and side eg are congruent to each other. side ab, side de, and side ef are congruent to each other. the length of side bc is 12 and the length of side cd is 10. the measure of angle bac is 50 degrees and the measure of angle dec is (5x-10) degrees. a. 2 < x < 5 b. 2 < x < 10 c. 2 < x < 12 d. 2 < x < 22

Answers

The inequality that describes the possible value of x is

c. 2 < x < 12

How to find the value of x

The triangles described gave the information to form the inequality equation

from the description it can be deduced that

m< bca = m< dce vertical angles

since side 12 is greater than 10 then m< bac > m< dec

therefore 50 > 5x - 10

solving for x

50 > 5x - 10

50 + 10 > 5x

60 > 5x

x < 12

therefore 2 < x < 12 is the inequality that represents x

Learn more about inequality at:

https://brainly.com/question/29244324

#SPJ1

1st question answer pls

 1st question answer pls

Answers

let's take a peek at the picture above, hmmm let's notice the vertex is at (-1 , 2), now let's get a point besides the vertex hmmm let's see it passes through (-2 , -1).

So we can reword that as what's the equation of a quadratic whose vertex is at (-1 , 2) and it passes through (-2 , -1)?

\(~~~~~~\textit{vertical parabola vertex form} \\\\ y=a(x- h)^2+ k\qquad \begin{cases} \stackrel{vertex}{(h,k)}\\\\ \stackrel{a~is~negative}{op ens~\cap}\qquad \stackrel{a~is~positive}{op ens~\cup} \end{cases} \\\\[-0.35em] ~\dotfill\)

\(\begin{cases} h=-1\\ k=2\\ \end{cases}\implies y=a(~~x-(-1)~~)^2 + 2\hspace{4em}\textit{we also know that} \begin{cases} x=-2\\ y=-1 \end{cases} \\\\\\ -1=a( ~~-2-(-1) ~~ )^2 + 2\implies -3=a(-2+1)^2\implies -3=a \\\\\\ ~\hfill~ {\Large \begin{array}{llll} y=-3(x+1)^2 + 2 \end{array}} ~\hfill~\)

Answer:

y = -3(x + 1)^2 + 2

Step-by-step explanation:

y = a(x - h)^2 + k is the vertex form of a quadratic, where

(x, y) are any point that lies on the parabola,a is a constant determining whether the parabola opens upward or downward,and (h, k) are the coordinates of the vertex.

Finding (h, k):

We see from the graph that the vertex is a maximum and its coordinates are (-1, 2).  Thus h is -1 and k is 2.  Since h becomes negative, it will be 1 in the parentheses: (x - (-1) = (x + 1).

Finding a:

In order to find a, we will need to plug in a point on the parabola for (x, y) and (-1, 2) for h and k.  We see that (0, -1) lies on the parabola so we can use this point for (x, y).

-1 = a(0 - (-1))^2 + 2

-1 = a(0 + 1)^2 + 2

-3 = a(1)^2

-3 = a

Thus, a = -3.

Thus, the exact equation in vertex form of the parabola is:

y = -3(x + 1)^2 + 2

I attached a picture from Desmos Graphing Calculator that shows how the equation I provided works and contains the two points you marked on the parabola, including (-1, 2) aka the maximum, and (0, -1) aka the y-intercept.

 1st question answer pls

Which statement regarding the equation below is true?
x²+y²-3 x+6 y=7 x+2
A. It has a diameter of 18 units.
B. It has an area of 36 π square units.
C. It has a center at (-5,3).
D. It has a center at (-2,-3).
A
B
C
D

Which statement regarding the equation below is true?x+y-3 x+6 y=7 x+2A. It has a diameter of 18 units.B.

Answers

The true statement regarding the equation x² + y² - 3x + 6y = 7x + 2 is that it has a area of 36π sq. unit ,  correct option is (b) .

In the question ,

it is given that ,

the equation is x² + y² - 3x + 6y = 7x + 2 ,

On simplifying and rewriting the equation ,

we get ,

x² - 10x + 25 + y² + 6y + 9 = 36

further it can be written as ;

(x - 5)² \(+\) (y \(+\) 3)² \(=\) 36

(x - 5)² + (y + 3)² = 6²

the equation represents a circle .

So , the from the above equation we can conclude that :

the center of the circle will be  = (5,-3) .

the radius is = 6 units .

the diameter will be = 12 units , and

the area of the circle will be = πr² = 36π square units .

So , the statement that matches with the information is that the area is 36π square units .  

Therefore , the True statement is (b) it has a area of 36π sq. units  .

Learn more about Circle here

https://brainly.com/question/9031206

#SPJ4

\(3-\sqrt{x} 5\)

Answers

The value of the expression given as 3 - √x⁵ is 3 - x^2√x

What are expressions?

Expressions are mathematical statements that are represented by variables, coefficients and operators

How to evaluate the expression?

The expression is given as

3 - √x⁵

Express 5 as the sum of 4 and 1

So, we have the following equation

3 - √x⁵ = 3 - √x^(4 + 1)

Apply the law of indices to expand the equation

So, we have the following equation

3 - √x⁵ = 3 - √(x^4 * x)

Next, we expand the equation

So, we have the following equation

3 - √x⁵ = 3 - √x^4 * √x

Evaluate the square root of x^4

So, we have the following equation

3 - √x⁵ = 3 - x^2 * √x

Evaluate the product

So, we have the following equation

3 - √x⁵ = 3 - x^2√x

Hence, the value of the expression is 3 - x^2√x

Read more about expressions at

https://brainly.com/question/4541471

#SPJ1

Use the diagram to complete the proportion. (Geometry)

FDBGEC

Use the diagram to complete the proportion. (Geometry) FDBGEC

Answers

Answer:

The answer is

13)CE

14)EG

15)BD

16)BF

Other Questions
What would happen if voltage gated K+ channels took longer thannormal to open? and how does this effect the amplitude, time tostart the falling phase, and maximum firing rate?Can please draw the ac 2. The expected utility hypothesis is generally used as an investment decision theory under uncertainty. Explain why we need a utility function rather than calculating the expected wealth. 3. Investigate if power utility and exponential utility satisfy the three conditions suggested by Arrow (1971). 4. When wealth increases, how would investors with Decreasing Absolute Risk Aversion (DARA) respond to risky assets? Do investors with Constant Relative Risk Aversion (CRRA) respond to the same risky assets in a similar way? In a parasitic relationship, _____.A. one organism benefits and the other is not harmedB. both organisms benefitC. one organism benefits and the other is negatively affectedD. neither organisms benefits{ this is for science} Crossing an individual who is homozygous dominant for a trait with an individual whose genotype is unknown will most likely produce which set of offspring?. Find the word that acts as either a predicate noun or predicate adjective in the sentence. Then tell which of the two it is.Your work is almost always neat. The sentence to the left, the word___ is a predicate___.A.is A.nounB.work B.adjectiveC.neatD.veryE.always What was the effect of the Jewish diaspora?OA. The Jewish people regained their homeland.B. The Jewish people in Judaea were all killed.C. The Jewish people left their homeland and spread to other lands.OD. The Jewish people were forced to abandon their religion in Ancient Rome. What are some ways in which a health care provider can show empathy while working with a diverse group of patients? hint: your answer might address diversity in age, sex, ethnicity, socioeconomic status, or others. this is for biomed science pltw and not for health answer the question !! How did Mars change once its interior became solid iron? Please help me with this homework DUE SOON!!! Who was Dr. Frederic Wertham and what was he trying to say about comic books? Does the same thing happen in todays world too? Pls help!! MARKING BRAINLIEST!!! Reflect on the section Planning and Executing Change Effectively, in Chapter 7 of the text. Summarize the key steps in planning and executing change in a 500 word journal entry Who proposed a model to describe the acculturation process of american ethnic minority populations, such as native americans? Which word in the sentence helps readers determine the meaning of the word? Calculate the magnitude of the couple created by the two forces shown below? 5) Given the cross section of a 7"x4" rectangular structural member as shown below, sketch the location of the centroidal x and y axes, indicate the location of the Centroid, and indicate which axis would have the larger Moment of Inertia (I). A 10 year old boy presents after his mother is concerned about a pigmented lesion appearing on his chin. On inspection, it is dome shaped, slightly raised and nodular, measuring 7mm diameter. It has a smooth surface. What is the most likely diagnosis? PC is purely a counter - Required input signals: Clock, Clr, Up - Required output signal: address to access instruction memory IR is purely a group of flip-flops which will latch out the input signal - Required input signals: Clock, ld, instruction from instruction memory - Required output signal: instruction to the finite state machine What is the maximum number of comparisons in a bubble sort if there are 5 elements in the array? 9. Who was the second president of the US? Who was his Vice President? Why was this the case?